Source: Veritas Prep

This topic has expert replies
Legendary Member
Posts: 1448
Joined: Tue May 17, 2011 9:55 am
Location: India
Thanked: 375 times
Followed by:53 members

Source: Veritas Prep

by Frankenstein » Fri Jul 15, 2011 2:46 am
Image

Please explain each answer choice.
OA is D
Expert comments would be appreciated!
Cheers!

Things are not what they appear to be... nor are they otherwise

Junior | Next Rank: 30 Posts
Posts: 29
Joined: Tue Jul 12, 2011 1:07 pm
Thanked: 8 times
Followed by:1 members

by beatthegmat.garry » Fri Jul 15, 2011 5:00 am
My guess was C as it says it a popular brand and may have become popular because of its goodwill among customers.
Not sure why the OA is D.

Thanks for posting though !

Legendary Member
Posts: 1448
Joined: Tue May 17, 2011 9:55 am
Location: India
Thanked: 375 times
Followed by:53 members

by Frankenstein » Fri Jul 15, 2011 5:23 am
Thanks for your response. Even, I selected C in the test :D
Cheers!

Things are not what they appear to be... nor are they otherwise

Master | Next Rank: 500 Posts
Posts: 189
Joined: Wed Jul 06, 2011 6:57 am
Thanked: 17 times
Followed by:1 members

by gmat25 » Fri Jul 15, 2011 6:46 am
Frankenstein wrote:Thanks for your response. Even, I selected C in the test :D
Guys, good to see i picked Op D only as my answer. Somewhere, The structure of the question is

X (media exposure) causes Y (popularity / strong standing in market)

still, Y happened without X

DISCREPANCY???

if some op suggest that X did happened and that argument misses this information and that's why Y happened, that gonna be good. Because that statement will prove X causes Y situation.
Op D, the OA did the same thing, hence correct

User avatar
GMAT Instructor
Posts: 2193
Joined: Mon Feb 22, 2010 6:30 pm
Location: Vermont and Boston, MA
Thanked: 1186 times
Followed by:512 members
GMAT Score:770

by David@VeritasPrep » Fri Jul 15, 2011 11:14 am
When I see the Veritas in the title I have to see what is being discussed!

Here we have a paradox question - so we have two things that should not happen at the same time.

One thing should be self-explanatory, the other will need to be explained.

In this case the statement that "companies simply cannot compete without a great deal of media exposure" is the self-explanatory fact. This does not need to be explained, just accepted as true.

The part that needs to be explained is how this company is able to compete when they do not advertise on radio or television.

Of course there is a switch in terms here "media exposure" is what is required. The lack of radio and tv advertising does not mean that there cannot be media exposure in some other way.

Answer Choice D goes just to this point exactly. They are getting free exposure on news shows. Another answer that would have worked is "they do lots of print ads in magazines and billboards and signs on buses and mass transit trains." This would give exposure and is not radio and tv.

Choice C makes the common mistake of trying to DENY one side of the paradox. You are not allowed to do this. If I tell you that all good cars are over $15,000 new and then I tell you that I got a good car for $10,000 how did it happen? Well the key word is NEW so I must have gotten a used car for 10,000 that would have been 15,000 + new. What I cannot do is suddenly say, well some new cars have a discount or something like this. You cannot deny either side of the paradox.

Hope it helps!
Veritas Prep | GMAT Instructor

Veritas Prep Reviews
Save $100 off any live Veritas Prep GMAT Course

Legendary Member
Posts: 1448
Joined: Tue May 17, 2011 9:55 am
Location: India
Thanked: 375 times
Followed by:53 members

by Frankenstein » Fri Jul 15, 2011 11:26 am
Hi,
When I see the Veritas in the title I have to see what is being discussed!
I have actually PMed you to discuss C and D. But, I came to know from your reply that you checked my PM after postign here.
In this case the statement that "companies simply cannot compete without a great deal of media exposure" is the self-explanatory fact. This does not need to be explained, just accepted as true.
I missed this point. Thanks a lot for clarifying this.
Cheers!

Things are not what they appear to be... nor are they otherwise

User avatar
Legendary Member
Posts: 1101
Joined: Fri Jan 28, 2011 7:26 am
Thanked: 47 times
Followed by:13 members
GMAT Score:640

by HSPA » Fri Jul 15, 2011 7:47 pm
David,
The last line.. " You cannot deny either side of the argument " . This is very new for me.
Is this true for all resolve the paradox type questions??


David@VeritasPrep wrote:When I see the Veritas in the title I have to see what is being discussed!

Here we have a paradox question - so we have two things that should not happen at the same time.

One thing should be self-explanatory, the other will need to be explained.

In this case the statement that "companies simply cannot compete without a great deal of media exposure" is the self-explanatory fact. This does not need to be explained, just accepted as true.

The part that needs to be explained is how this company is able to compete when they do not advertise on radio or television.

Of course there is a switch in terms here "media exposure" is what is required. The lack of radio and tv advertising does not mean that there cannot be media exposure in some other way.

Answer Choice D goes just to this point exactly. They are getting free exposure on news shows. Another answer that would have worked is "they do lots of print ads in magazines and billboards and signs on buses and mass transit trains." This would give exposure and is not radio and tv.

Choice C makes the common mistake of trying to DENY one side of the paradox. You are not allowed to do this. If I tell you that all good cars are over $15,000 new and then I tell you that I got a good car for $10,000 how did it happen? Well the key word is NEW so I must have gotten a used car for 10,000 that would have been 15,000 + new. What I cannot do is suddenly say, well some new cars have a discount or something like this. You cannot deny either side of the paradox.

Hope it helps!
First take: 640 (50M, 27V) - RC needs 300% improvement
Second take: coming soon..
Regards,
HSPA.

User avatar
GMAT Instructor
Posts: 2193
Joined: Mon Feb 22, 2010 6:30 pm
Location: Vermont and Boston, MA
Thanked: 1186 times
Followed by:512 members
GMAT Score:770

by David@VeritasPrep » Sat Jul 16, 2011 3:36 am
Certainly it is true of all paradox questions! It is the essence of a paradox.

If I told you that "people on average gain 1 pound per year after the age of 35 (which is not healthy for them), but then I told you that those who do not gain the weight actually die sooner" I have to keep both sides of this. I am not able to all of the sudden say that "gaining weight is actually good because it prevents dehydration and starvation."

This CAN NEVER be the answer, because it denies that the gaining weight is not healthy - something that has already been stated.

By the way, this is an LSAT question, I wonder if anyone knows the answer (just in words, without answer choices) to explain the paradox as to why the people who maintain the healthier weight die sooner?
Veritas Prep | GMAT Instructor

Veritas Prep Reviews
Save $100 off any live Veritas Prep GMAT Course

User avatar
Legendary Member
Posts: 1101
Joined: Fri Jan 28, 2011 7:26 am
Thanked: 47 times
Followed by:13 members
GMAT Score:640

by HSPA » Sat Jul 16, 2011 6:04 am
Hi David,

I guess those guys are doing over diet, natural metabolism makes you put up 1 pound, not gaining weight means excessive diet => health deprecation.
First take: 640 (50M, 27V) - RC needs 300% improvement
Second take: coming soon..
Regards,
HSPA.

User avatar
GMAT Instructor
Posts: 2193
Joined: Mon Feb 22, 2010 6:30 pm
Location: Vermont and Boston, MA
Thanked: 1186 times
Followed by:512 members
GMAT Score:770

by David@VeritasPrep » Sat Jul 16, 2011 7:07 am
Good thought! Not quite...

The LSAT is obsessed with one thing -- smoking. Weird, huh?

So the answer is that the people who do not gain weight are those who smoke and this prevents the weight gain but causes other problems.

Your answer is very good, but even this idea that they are too strict with the diet would slightly deny the fact gaining weight is bad. The smoking answer does not deny that gaining weight is bad. It just says that the smoking is worse!
Veritas Prep | GMAT Instructor

Veritas Prep Reviews
Save $100 off any live Veritas Prep GMAT Course

User avatar
Master | Next Rank: 500 Posts
Posts: 123
Joined: Tue May 31, 2011 12:26 am
Location: Hyderabad
Thanked: 5 times
Followed by:1 members

by jainnikhil02 » Sat Jul 16, 2011 7:47 am
i also choose D because no other option was best among all.
But Thanks David.. ur explanation always ROCKS... Keep it up..

Thanks a lot
Nikhil K Jain
____________________

"Life is all about timing" Don't waste your and others time.

User avatar
Senior | Next Rank: 100 Posts
Posts: 35
Joined: Fri Jul 23, 2010 5:09 am
Thanked: 1 times
Followed by:1 members

by vishwas.arora » Wed Jul 20, 2011 11:41 am
IMO D